proving the limit of a product is the product of the limits, epsilon-delta definition

Sdílet
Vložit
  • čas přidán 16. 07. 2024
  • We will prove the limit of a product is the product of the limits (assuming the limits exist) by using the epsilon-delta definition of a limit. This is a classic proof that you will see in your advanced calculus class or real analysis class. This is hard proof especially when you first see it. Be sure you put in the effort and study it well. #calculus #realanalysis #blackpenredpen
    Notes for this video are available on my Patreon: 👉 / notes-proof-of-82385985
    Introduction and more examples of the epsilon-delate definition of a limit: 👉 • Epsilon-Delta definiti...
    0:00 Review some limits and their epsilon-delta definition
    2:48 The proof!
    🛍 Shop math t-shirts & hoodies: blackpenredpen.creator-spring...
    10% off with the code "WELCOME10"
    ----------------------------------------
    💪 Support the channel and get featured in the video description by becoming a patron: / blackpenredpen
    AP-IP Ben Delo Marcelo Silva Ehud Ezra 3blue1brown Joseph DeStefano
    Mark Mann Philippe Zivan Sussholz AlkanKondo89 Adam Quentin Colley
    Gary Tugan Stephen Stofka Alex Dodge Gary Huntress Alison Hansel
    Delton Ding Klemens Christopher Ursich buda Vincent Poirier Toma Kolev
    Tibees Bob Maxell A.B.C Cristian Navarro Jan Bormans Galios Theorist
    Robert Sundling Stuart Wurtman Nick S William O'Corrigan Ron Jensen
    Patapom Daniel Kahn Lea Denise James Steven Ridgway Jason Bucata
    Mirko Schultz xeioex Jean-Manuel Izaret Jason Clement robert huff
    Julian Moik Hiu Fung Lam Ronald Bryant Jan Řehák Robert Toltowicz
    Angel Marchev, Jr. Antonio Luiz Brandao SquadriWilliam Laderer Natasha Caron Yevonnael Andrew Angel Marchev Sam Padilla ScienceBro Ryan Bingham
    Papa Fassi Hoang Nguyen Arun Iyengar Michael Miller Sandun Panthangi
    Skorj Olafsen Riley Faison Rolf Waefler Andrew Jack Ingham P Dwag Jason Kevin Davis Franco Tejero Klasseh Khornate Richard Payne Witek Mozga Brandon Smith Jan Lukas Kiermeyer Ralph Sato Kischel Nair Carsten Milkau Keith Kevelson Christoph Hipp Witness Forest Roberts Abd-alijaleel Laraki Anthony Bruent-Bessette Samuel Gronwold Tyler Bennett christopher careta Troy R Katy Lap C Niltiac, Stealer of Souls Jon Daivd R meh Tom Noa Overloop Jude Khine R3factor. Jasmine Soni L wan na Marcelo Silva Samuel N Anthony Rogers Mark Madsen Robert Da Costa Nathan Kean Timothy Raymond Gregory Henzie Lauren Danielle Nadia Rahman Evangline McDonald Yuval Blatt Zahra Parhoun Hassan Alashoor Kaakaopuupod bbaa Joash Hall Eric
    ----------------------------------------
    Thank you all!

Komentáře • 198

  • @blackpenredpen
    @blackpenredpen  Před rokem +23

    Notes for this video are available for my patrons: 👉 www.patreon.com/posts/notes-proof-of-82385985

    • @annaclarafenyo8185
      @annaclarafenyo8185 Před rokem

      This video is nonsense. The proof that the multiplication function is continuous is trivial, and the proof that the composition of continuous functions is continuous is also trivial. (x+epsilon)(y+delta) = x y + epsilon*x + delta*y +epsilon*delta, and all quantities are infinitesimal aside from x*y. This observation can be turned into an epsilon delta proof automatically by rote. It's ridiculous to call this difficult, it's obvious.

    • @blackpenredpen
      @blackpenredpen  Před rokem

      @@annaclarafenyo8185 f and g might be discontinuous

    • @annaclarafenyo8185
      @annaclarafenyo8185 Před rokem

      @@blackpenredpen Continuous AT THE POINT means the limit of f equals M and the limit of g equals N, you can set the values to be M and N, and then they are continuous, and the continuous composition of continuous functions is continuous. These methods are formalizations of infinitesimal arguments that are obvious: (f+df)(g+dg)=fg + infinitesimal. There is no work in formalizing this. Making this obvious nonsense sound difficult just serves to mystify epsilon-delta, and make it a hazing ritual rather than a method of proof you internalize.

    • @blackpenredpen
      @blackpenredpen  Před rokem

      @@annaclarafenyo8185 f and g don’t need to be continuous at a

    • @annaclarafenyo8185
      @annaclarafenyo8185 Před rokem

      @@blackpenredpen IF YOU DEFINE THEM SO THAT THE LIMIT IS EQUAL, THEY ARE BY DEFINITION CONTINUOUS AT A. You should not be teaching matheamatics.

  • @what_a_lame_tag_system
    @what_a_lame_tag_system Před rokem +334

    You know something is wrong when bprp doesn't use whiteboard for proof

    • @andreasxfjd4141
      @andreasxfjd4141 Před rokem +6

      Simply if he doesn’t use a whiteboard (than something has changed)

    • @akasunanosasori7547
      @akasunanosasori7547 Před rokem +5

      Actually my teacher showed us this proof in grade 11, I don’t think it’s too hard (now we weren’t supposed to learn the proof but showed us nonetheless)

    • @blackpenredpen
      @blackpenredpen  Před rokem +55

      I needed to keep everything on the board for this video thus I chose the iPad route 😆

    • @Paul-ob2hy
      @Paul-ob2hy Před rokem +2

      @@blackpenredpenwhat app did you use on your ipad? would it be good for notes at university?

    • @blackpenredpen
      @blackpenredpen  Před rokem +16

      @@Paul-ob2hy I never took notes on an iPad when I was a student. However, I have been using the app "Good Notes" and I have been loving it!

  • @_4y4m3_ch4n_
    @_4y4m3_ch4n_ Před rokem +106

    27:05 to avoid the 0/0 case, we can simply have epsilon*|L|/[2(|L| + 1)] < epsilon*(|L| + 1)/[2(|L| + 1)] = epsilon/2, and the solution is unaffected in any way ^^

  • @somerapdude
    @somerapdude Před rokem +14

    Not to kiss your behind or whatever, but this is the most helpful channel I’ve ever found when it comes to helping me with my EXTREMELY hard math class

  • @CallMeIshmael999
    @CallMeIshmael999 Před rokem +11

    This is one of the more complicated proofs of basic analysis facts and I've never really had a good intuition for the quantities that come up in the proof, so thank you for doing a good job of talking through it.

  • @epsilia3611
    @epsilia3611 Před rokem +13

    26:07 I saw some people trying to explain how we don't have to bother about the |L|=0 case in a certain way, but in the end :
    We know that |L| < [L|+1 for any real number L. So |L|/(|L|+1) < 1 as a consequence. Multiplying each sides by eps/2 (> 0) guarantees us the result !
    Edit : Nevermind it has been said by multiple people already, my bad

  • @thatonemailbox
    @thatonemailbox Před rokem +29

    1:28 This is the best explanation of the epsilon-delta definition of limits I have ever seen. Everything for me just clicked once I saw this. You're an awesome teacher!

  • @Maths_3.1415
    @Maths_3.1415 Před rokem +10

    This is blackpenredpenbluepengreenpen and yellow highlighter

  • @tzovgo
    @tzovgo Před rokem +19

    26:27 You don't have to cancel two zeros if you add 1 to |L|, proving that step rigorously!
    |L| * (epsilon)/(|L|+1) < (|L| + 1) * (epsilon)/(|L|+1) = (epsilon)/2

    • @keedt
      @keedt Před rokem +1

      was going to write this 👍.

    • @blackpenredpen
      @blackpenredpen  Před rokem +4

      Ah yes. I didn’t think of that. Thanks for letting me know.

    • @Apollorion
      @Apollorion Před rokem

      tzovgo, you made a little mistake in the formula, which I didn't notice at first, neither did, I presume, BPRP. We both understood what you meant, but you forgot to put the 2 in the denominator whenever |L| was present in the quotient. This is what it should be:
      epsilon*|L|/[2(|L| + 1)] < epsilon*(|L| + 1)/[2(|L| + 1)] = epsilon/2

  • @yohangross5518
    @yohangross5518 Před rokem +2

    extremely good job and explanations. As a new online teacher, I find your content really inspiring. I now recognize that I mimic a lot your teaching style. Keep up the good job !

    • @blackpenredpen
      @blackpenredpen  Před rokem +1

      Thank you for your kind words. Wishing you all the best! : )

  • @mariannelee9594
    @mariannelee9594 Před rokem +3

    I'm graduating soon in Mechanical Engineering and I just have to say your channel is the best! You've helped me so much!

  • @titan1235813
    @titan1235813 Před rokem +1

    Bprb, WHAT AN AWESOME PROOF! Even though it turned out to be a very difficult one, I understood every step of it, and that's because you are en EXCELLENT TEACHER. Thank you! 🙏🏻

  • @martinepstein9826
    @martinepstein9826 Před rokem +2

    I prefer to use epsilon-delta arguments only in the simplest cases. For example,
    - A sum of two functions approaching 0 approaches 0
    - A function approaching 0 times a bounded function approaches 0
    Now f can be expressed as L plus a function approaching 0, and g can be expressed as M plus a function approaching 0. You can just multiply it out and see that the product is LM plus a function approaching 0 i.e. the limit of the product is LM.

  • @yanceyward3689
    @yanceyward3689 Před rokem

    I have always loved working out these fundamental proofs. I actually did this particular one out just a few months ago working through my Schaum's Calculus workbook. I didn't think it was particularly difficult- the really key insight is that addition/subtraction trick that allows the breaking apart of the functions, but it is a common technique in algebra, so came to me almost instantly.

  • @cgandcats
    @cgandcats Před 14 dny

    Thank you very much!!! I Found this proof in Spivak's book but I was not getting how it was done and after watching your video now is all clear to me. Love you!

  • @michaelbaum6796
    @michaelbaum6796 Před rokem

    Excellent explanation 👍

  • @scottleung9587
    @scottleung9587 Před rokem +1

    Wow - this takes me back to my modern analysis course in college. I'll never forget how brutal that was. But good job with the proof!

  • @jvthunder6548
    @jvthunder6548 Před rokem +16

    I remember getting this question in my university practice and getting the proof myself is so satisfying!

  • @darkdelphin834
    @darkdelphin834 Před rokem +1

    You are looking CLEAN on the thumbnail man. Nice

  • @srikanthtupurani6316
    @srikanthtupurani6316 Před rokem +2

    Main trick is writing g(x) as
    g(x) -M+M after this there is no need to worry about the term f(x) g(x). It is all about being careful. We have to adjust things.

  • @marble17
    @marble17 Před rokem +1

    Seeing math makes me remembers when i find the answer
    Like "I have 9 numbers, the first 5 numbers has a mean of 12, while the last 4 numbers has a mean of 3, what is the mean of all 9 numbers"
    And thats when i realise i can just do (5 × 12 + 4 × 3) ÷ 9 and got 8

  • @henrybarber288
    @henrybarber288 Před rokem +1

    You could also just prove the rule for limits of sequences, and then use the sequential definition of the limit to get the same result for limits of functions.
    Another approach would be to prove the result in the special case L = 0 = M. This is quite simple as you just choose δ such that
    |x - a| < δ ⇒ |f(x)| , |g(x)| < √ε
    Then for the more general case define new functions:
    F = f - L, G = g - M
    Now the limit of both F and G is just zero, so the limit of FG is zero. Do a bit of expanding and out comes that the limit of fg is LM.

  • @qianxiaogouQy
    @qianxiaogouQy Před 9 měsíci +1

    Thank u so muchhhhhhhhhhh!!!!!!!!! U save me life by this wonderful video, thank uuuuuuuuuuuuuuuuuuuuuuuuuu!!!!!!!!!!!!!!

  • @chahine__8296
    @chahine__8296 Před 3 měsíci

    Amazing video, thank you so much

  • @maelhostettler1004
    @maelhostettler1004 Před rokem

    I find it easier in the general case of a finite dimension vectorial space like R^n and considering a subordinate norm which is sub-multiplicative... Of course you need a bit of topology but the proof is just much nice

  • @piotrskalski1477
    @piotrskalski1477 Před rokem +1

    I know the one for sequences and it is quite hard. I had to make my own one because I couldn't remember the one from the lecture. Anyway, from that you can easily arrive at the one for functions thanks to the sequence definition of a limit

  • @feuerrm
    @feuerrm Před rokem +1

    Flashbacks to Real Analysis I where our professor wanted us to figure out how to do this proof on our own *shudders*

  • @BedrockBlocker
    @BedrockBlocker Před rokem +1

    That is why you derive the sequence criterion for limits, and then you can use the easier to prove limit theorems.

  • @rmlu9767
    @rmlu9767 Před rokem +2

    I think the easiest way to think about the 0/0 part is: if |L|=0, then |L|*epsilon/(2*(|L|+1))=0 which is definitely smaller than epsilon/2.

    • @Apollorion
      @Apollorion Před rokem +1

      Not greater, smaller: 0 is the smallest non-negative real number!

    • @rmlu9767
      @rmlu9767 Před rokem

      @@Apollorion thank you. Corrected.

  • @ignassablinskas9175
    @ignassablinskas9175 Před 10 měsíci

    Great explanation.

  • @ahlamouldkhesal5562
    @ahlamouldkhesal5562 Před rokem +1

    Fun fact we in Algeria we study real analysis in the first year of bachelor degree

  • @dulot2001
    @dulot2001 Před rokem +1

    You can simplify the proof by using the lemma :
    lim_(x->a)f(x)=L iff there exists K>0 s.t. for all epsilon >0 there exists delta s.t |x-a| |f(x)-L|

  • @justinyoung4381
    @justinyoung4381 Před rokem +4

    Alternative idea: Prove that the multiplication map (x,y) -> xy, is continuous R^2 -> R, which isn't that hard. Then, the result follows easily.

    • @justinyoung4381
      @justinyoung4381 Před rokem +2

      On second thought, it's more or less the same as your proof, but seems simpler because of the removal of f(x) and g(x).

  • @aosidh
    @aosidh Před rokem

    I was just trying to remember this proof! (with some difficulty after ~15 years 😅)

  • @mcalkis5771
    @mcalkis5771 Před rokem +1

    This video is filled with tasty _rigor_
    I love it...
    Make more of these please

  • @joshmckinney6034
    @joshmckinney6034 Před rokem +1

    Omg I can’t believe I actually understood all of this!

  • @theoriginyt4869
    @theoriginyt4869 Před rokem +5

    Now proof that the proof is 1618 times harder than the calculation

    • @Apollorion
      @Apollorion Před rokem +1

      ..and not 1619 or 1617 times harder.

  • @aryankushwaha7028
    @aryankushwaha7028 Před rokem +2

    Sir if we have to prove cone curve surface area then its can be prove by like this ,we take a triangle in it and move about dy angle from central then on cover surface area there would be a thin triangle and bease would be r× dy(it approximately be taken as straight line in calculus)and height lateral height L then area of that triangle would be 1/2×r×dy×L and when we intergrate it with limits 0 to 2pie our answer is pie ×r×L, sir does this is correct , and dy×r is straight line

  • @eduardvelasco1
    @eduardvelasco1 Před rokem

    The example needs not the propiety, the function is continous therefore the limit converges to the value

  • @tunafllsh
    @tunafllsh Před rokem

    Using 1 is nice. I used to introduce another constant something epsilon2

  • @orenfivel6247
    @orenfivel6247 Před rokem

    PF of "as x→a, f(x)/g(x)→L/M" directly is also "hard".
    "directly" i mean without proving the special case "as x→a, 1/g(x)→1/M" and using prada law for f(x)/g(x)=f(x)*[1/g(x)]

  • @rakrius7839
    @rakrius7839 Před rokem

    Do more proofs please. ❤

  • @phantienminhthuy3805
    @phantienminhthuy3805 Před 4 měsíci +1

    awesome!!

  • @armanavagyan1876
    @armanavagyan1876 Před rokem

    I liked how U say ah ah fells so good😂😂😂

  • @abhinavraj4845
    @abhinavraj4845 Před rokem +8

    Woahh......I feel smarter

  • @theproofessayist8441
    @theproofessayist8441 Před rokem +2

    Zero pairs are evil period for me - this is why I liked when Dr Peyam did the proof for derivative of product rule he drew a bunch of quadrilaterals within quadrilaterals and subtracted them out. - only caveat is you need to suspend disbelief that this does not work just for positive but negative area (and functions as well).

  • @Maths_3.1415
    @Maths_3.1415 Před rokem +4

    Please make a tutorial on how you switch your markers on this board

    • @epsilia3611
      @epsilia3611 Před rokem

      I didn't know you were a genius in humour too mister

    • @Maths_3.1415
      @Maths_3.1415 Před rokem

      ​@@epsilia3611 thank you 😅

    • @blackpenredpen
      @blackpenredpen  Před rokem +1

      waiting for someone to buy me another apple pencil lol

    • @Maths_3.1415
      @Maths_3.1415 Před rokem +1

      ​@@blackpenredpen 😂

  • @cruzcostilla9796
    @cruzcostilla9796 Před rokem

    Hi Blackpenredpen. Make a video demonstrating why π is irrational. I am subscribed to your channel, very good videos. Greetings from Mexico

  • @yanceyward3689
    @yanceyward3689 Před rokem

    Here is how I broke up the functions:
    |f(x)g(x)-LK| < ε3
    Add and subtract 2LK, and add and subtract [Kf(x) + Lg(x)] to get
    |f(x)g(x)-Kf(x)-Lg(x)+LK+Kf(x)-LK+Lg(x)-LK| < ε3
    |[f(x)-L][g(x)-K]+K[f(x)-L]+L[g(x)-K]|< ε3
    From here, you break it apart using the triangle inequality which gives eventually
    |f(x)g(x)-LK|< ε1ε2 + |K|ε1 + |L|ε2

  • @decare696
    @decare696 Před rokem

    Using nonstandard analysis, this theorem becomes quite easy to prove:
    Start with lim(x->a) f(x)g(x) =: A.
    For any nonzero infinitesimal h, we have
    A = st(f(a+h)g(a+h)).
    But since
    L = lim(x->a) f(x) = st(f(a+h)) and
    M = lim(x->a) g(x) = st(g(a+h))
    for any nonzero infinitesimal h, there are infinitesimals k,l for which
    f(a+h) = L + k and
    g(a+h) = M + l.
    This directly implies
    A = st((L+k)(M+l)) = st(LM +kM + lL + kl) = LM.
    QED

  • @blackplays4452
    @blackplays4452 Před rokem

    I feel like its kind of the engineer way how you explain the first example. A (for me nicer way of justifying this is by setting: h(x)=f(x)g(x), and assume they are continuous, which implies, that the limit for all a (exept infinity) exists. Now we can use sequence continuity by saying: lim ( f(x)g(x) )= lim h(x)=h( lim x ) = f( lim x )g( lim x). And we are done. I am of course assuming that you need to know about sequence continuity, but this should be more elementary than limit to some random point a, where you often have some 0/0 or infinity/infinity problems :)

  • @armanavagyan1876
    @armanavagyan1876 Před rokem

    Thanks PROF)

  • @afrolichesmain777
    @afrolichesmain777 Před rokem +6

    26:30 I think it would be a clearer explanation to simply state that for all values of L, |L| < |L| + 1. Diving both sides by |L| + 1 gives [ |L| / (|L| +1) ] < 1, so you have [ |L|*eps / 2*(|L| + 1) ] < eps/2. Great video! Haven’t seen this proof since my undergrad in 2018.

    • @Ninja20704
      @Ninja20704 Před rokem +2

      That’s what i was thinking too. If we regroup the terms as
      [eps/2][|L|/|L|+1]

  • @alevelmathsmastery
    @alevelmathsmastery Před rokem +1

    Next video: 100 matrix transformation and eigenvalue questions

  • @lychenus
    @lychenus Před rokem +2

    this is very easily taught in asia. its just that NA students arent exposed to inequality too much.

  • @silver6054
    @silver6054 Před rokem +2

    Back in the day when I used to have to do such proofs, I was irritated by the need to "edit" the proof after completing it, so choosing epsilon/3 for example, as the last line added three such terms. It makes the proof more "magical" for those trying to read it ("Why epsilon/3!!!!") After all, anyone who understands the limit/continuity/whatever definitions understands that "< 3*epsilon" is as good as "< epsilon" given the arbitrary choice of epsilon in the first place.

    • @theproofessayist8441
      @theproofessayist8441 Před rokem

      Lol I was lazy and sometimes said you see how all the lines of deduction are in order when thinking from scratch and stuff - told the professor in comments please read from bottom to top in reverse. They said no - you have to rewrite the entire proof again. I found that was annoying.

  • @shashikr814
    @shashikr814 Před rokem

    Can you make Differentiation and integration basics concepts for beginners.
    I'm unable to understand it in my school.

  • @gbessinpenieleliezerhoumba3337

    Hi dear Professor, I have a topic that might interest you and I would like to see a video about it:
    PROVE THAT FOR EVERY REAL NUMBER NOT A MULTIPLE OF 2 CONTAINS IN ITS DECOMPOSITION AT LEAST ONE ODD NUMBER

  • @JayTemple
    @JayTemple Před rokem +2

    Once you have delta-1 and delta-2, do you even need delta-3? That is, can’t you take min(delta1, delta2) and leave it at that?

    • @blackpenredpen
      @blackpenredpen  Před rokem +1

      Yes, bc I used the given limits for three different ineq in the blue part.

  • @gytoser801
    @gytoser801 Před rokem

    What is the name of the whiteboard program??

  • @tunafllsh
    @tunafllsh Před rokem

    I remember when I first watched bprp I was in high school and didn't know many things. Now I can be more rigor than bprp.

  • @sebgor2319
    @sebgor2319 Před 11 měsíci

    Could you prove that integral of e^(-x^2) has no solution in elementary functions?

  • @amr0733
    @amr0733 Před rokem

    try finding the values for a, b and c when
    f(x)=ax^2+bx+c
    f(A)=B
    f(C)=D
    f(E)=F
    I did this equation myself and the answer is very long

  • @Ferraco05
    @Ferraco05 Před rokem +1

    The last step can be easily justified by noticing
    ε|L|/[2(|L|+1)] = (ε/2)[|L|/(|L|+1)]
    and also
    |L|/(|L|+1) < 1
    This is true even when L=0. Therefore
    ε|L|/[2(|L|+1)] < ε/2
    QED lol
    Edit: I realized afterwards many others explained this already or similar arguments haha but yeah the proof is sound, was really nice following it till the end

  • @happy.5
    @happy.5 Před rokem

    Give suggestion to study PDE , Its hard to grasp its concepts

  • @lawrencejelsma8118
    @lawrencejelsma8118 Před rokem

    I don't remember L'Hopital's Rule being so difficult needing f(x)/h(x) when there is a numerator and denominator limit of the form f(x>a)g(x>a) 😬🤣

  • @calculus988
    @calculus988 Před rokem +2

    Blackpenredpen can you do a proof of descartes rule of signs. I'm starving for that proof. You would explain the proof very easily. 😢

  • @tapu_
    @tapu_ Před rokem +1

    1+1=2 also hard to prove if you look in uni books

  • @ChaoticMagnet
    @ChaoticMagnet Před rokem +1

    How about:
    dy/d(dy/dx) = y

  • @jimallysonnevado3973
    @jimallysonnevado3973 Před rokem

    if |L| = 0 the expression is 0 and is definitely less than epsilon/2 (because epsilon is positive and half of it is still positive), otherwise |L|/(|L|+1) is less than 1 so the second expression is also less than epsilon/2.

  • @sharadvyas2025
    @sharadvyas2025 Před rokem

    I'll ask you a question
    What is -ln(-1) =?
    A . iπ
    B . -iπ
    C. A ans B both
    D. Can't possible
    E. None of the above
    I am not testing you but by watching just your videos i got that question in my mind

  • @blakedylanmusic
    @blakedylanmusic Před rokem +1

    When I did an online real analysis class the professor only proved this for limits of sequences. Then after that he didn’t bother proving it explicitly for limits of functions, rather he used the sequential characterization of limits to be like “we did this already” lol. I don’t blame him for not wanting to go through this again honestly!

    • @blakedylanmusic
      @blakedylanmusic Před rokem +1

      Also for the |L|/(1+|L|) thing, you could also break it into cases, ie if |L| = 0 then of course ε|L|/2(1+|L|) = 0 < ε/2. Then in the case |L| is nonzero you do what you showed in the video.

    • @blackpenredpen
      @blackpenredpen  Před rokem +1

      In fact, I should have done it as (like many people who have already pointed out)
      |L|/(|L|+1) < (|L|+1)/(|L|+1) = 1
      I couldn't believe I didn't think of that when I was working this out lol

    • @blakedylanmusic
      @blakedylanmusic Před rokem

      @@blackpenredpen honestly as a grad student in math, I feel like that a lot 😂😂😂 I’ll spend hours trying to come up with a certain proof and then when I look at my professors solution I’m like “why didn’t I think of that” hahaha

  • @serae4060
    @serae4060 Před rokem +1

    Let ɛ < 0

  • @ahlamouldkhesal5562
    @ahlamouldkhesal5562 Před rokem

    We allwayse proof the limits using the defenition of limits

  • @MasterHigure
    @MasterHigure Před rokem +1

    0:45 Does the limit of x as x approaches 3 exist? Yes, it does. How do we know? Epsilon-delta time!

  • @tobybartels8426
    @tobybartels8426 Před rokem +2

    This is why we have these theorems in the first place, right? You don't want to go through this ε-δ stuff every single time, so you go through it *once,* to prove the theorem, and afterwards just use the theorem.

    • @tobybartels8426
      @tobybartels8426 Před rokem

      @LeftRight : That's true, but even those are closer to the general proof for multiplication that we did, than to the specific proof for x√(x+1) that we avoided. (Besides, if we really prove all the theorems that calculus students implicitly rely on, for addition, multiplication, powers, trig functions, etc, then we'll have gotten plenty of practice.)

  • @aakifrehman8150
    @aakifrehman8150 Před rokem +2

    What videos is he referring to, where he has explained the epsilon-delta.

    • @Theraot
      @Theraot Před rokem

      czcams.com/video/DdtEQk_DHQs/video.html

    • @blackpenredpen
      @blackpenredpen  Před rokem

      See links in description 😃

  • @sitienlieng
    @sitienlieng Před rokem

    Nice proof! We can also let |g(x)-M|

  • @tunafllsh
    @tunafllsh Před rokem

    27:00 this part is simple. Notice |L|/(|L|+1) < 1

  • @davidbrisbane7206
    @davidbrisbane7206 Před rokem +4

    Great explanation. Technical requirements explained so well that a 14 year old could understand it 👍.

  • @dipun4849
    @dipun4849 Před rokem

    Sir,I try this as a undergraduate(12th) student but I can't. And also want to prove derivative of u/v and uv rule. Love from India.

  • @MathOrient
    @MathOrient Před rokem +1

    👍

  • @kodadmrx5859
    @kodadmrx5859 Před rokem +1

    Please integrate this hard problem :
    İntegral ln(x)ln(1-x)/(1+x²) dx borders 0 to 1 😓😓

  • @boltez6507
    @boltez6507 Před rokem +2

    I always though this was obvious,and therefore non provable😅

  • @JM-us3fr
    @JM-us3fr Před rokem

    Now do the Chain Rule 😅

  • @Ninja20704
    @Ninja20704 Před rokem

    Can I ask, how do we use the epsilon-delta defn to prove a limit than involves x-> +/- infinity? Because infinity is not a number so we cant say a=+/-inf. Or is there a different definition we have to use?
    Thank you in advance

    • @thatapollo7773
      @thatapollo7773 Před rokem

      I will give an example
      Say we need to prove lim x->+inf 1/x = 0
      We need to show that for all epsilon > 0
      There exists delta st. For all x>delta(THE KEY DIFFRENCE)
      f(x) - 0 < epsilon

    • @SatanicNerfd
      @SatanicNerfd Před rokem

      One way to see this is by substituting y=1/x and take the limit y->0

    • @Ninja20704
      @Ninja20704 Před rokem

      @@thatapollo7773 Thank you. But it should be |f(x)-0|

    • @vonneumann6161
      @vonneumann6161 Před rokem +1

      Yes, there are different definitions for all of those. For example, if you want to prove lim[x->a]f(x) = +infinity, you have to show that for any M there exists some delta>0 such that if |x-a| < delta then f(x) > M. It’s similar to epsilon-N definition. It’s a bit different when a = +/- infinity and all the other versions but they’re all basically the same.

    • @thatapollo7773
      @thatapollo7773 Před rokem

      ​@@Ninja20704 yes, I forgot to put the absolute value

  • @armanavagyan1876
    @armanavagyan1876 Před rokem

    PROF thinked this better than blackboard.

  • @justkarl2922
    @justkarl2922 Před rokem

    There is a subtle mistake because just because the function f and g converge to certain values L and M that doesn't mean there continous only if the value of the function is the same as the limit so L should be f(a) and M=g(a) but okay. shorter proof: Because f and g are continous they're bounded in a little neighbourhood around "a". We can find a bounding constant for both of them: ∃M>0: |f|,|g|0 and because f and g are supposed as continous functions we find a delta for both of them such that |f(x)-f(a)|

  • @pauselab5569
    @pauselab5569 Před 10 měsíci

    tried that during class. wasted 30 minutes, not sure if my proof works, there might be an easy proof that doesn't even rely on delta epsillon:(. guess imma post my proof on reddit and hope for the best.

  • @siraj522
    @siraj522 Před rokem

    way better than whiteboard

  • @hatnoob7068
    @hatnoob7068 Před rokem

    Next video: Polynomial division?

  • @mgancarzjr
    @mgancarzjr Před rokem

    "The proof is left as an exercise for the professor."

  • @christophniessl9279
    @christophniessl9279 Před rokem

    If |L| = 0 then obviously |L|*|g(x)-M| =0 since this is 0 multiplied with something. And 0 is always less than ε/2 since ε is positive. For |L| >0 the shown method works.

  • @user-yn1mu2eb8t
    @user-yn1mu2eb8t Před rokem

    1618 = 2×809

  • @Daniel-oy2he
    @Daniel-oy2he Před rokem

    😂Is that a Monty Python joke? 1618, a movie about the Spanish Inquisition?
    ETA: |L|/(|L|+1) < 1 since |L| < |L|+1. No shenanigans necessary. Great video.

    • @blackpenredpen
      @blackpenredpen  Před rokem +1

      No, 1618 is 1000*φ, the golden ratio 😃

    • @Daniel-oy2he
      @Daniel-oy2he Před rokem +1

      @@blackpenredpen Ah, nice. I didn't expect that or the Spanish Inquisition.

  • @tifn4g190
    @tifn4g190 Před rokem +3

    for me the worst in maths is when you have to prove an evidence (1+1=2)... Great video by the way

  • @flowingafterglow629
    @flowingafterglow629 Před rokem

    Why do you always indicate that |x-a| > 0? Is that only to specify that x=/= a?

    • @Zephei
      @Zephei Před rokem

      Well, x ≠ a is equivalent to |x-a| > 0, which is part of the definition of a limit.

  • @indudubey2127
    @indudubey2127 Před rokem

    Bring a calculus intro video for a 8 grader

  • @AxelGuillemette
    @AxelGuillemette Před rokem

    This actually pretty a basic proof you could even do it in a normal Vector Space it's the same thing